Statistics- unbiased estimator #2

  • Thread starter Thread starter Roni1985
  • Start date Start date
  • Tags Tags
    Statistics
Click For Summary
The discussion revolves around finding the expected value of the minimum of a random sample from an exponential distribution. The participant initially calculated E[n*Ymin] but was unsure about the correctness of their answer, suspecting a possible error leading to a negative result. Clarification was provided on deriving the CDF and PDF for Ymin, emphasizing the independence of the sample and the correct application of the exponential distribution's properties. The correct approach involves using the relationship between the CDF and PDF of Ymin to derive its expected value. Ultimately, confirming that E[n*Ymin] equals theta would indicate that the estimator is unbiased.
Roni1985
Messages
200
Reaction score
0

Homework Statement


There is some question, I solved it but am not sure I got the right answer.

Let Y1, Y2... Yn be a random sample of size n from the pdf fY(y;\Theta= \frac{1}{\Theta}*e-y/\Theta , y>0
Let \Theta_hat=n*Ymin is tex]\Theta[/tex]_hat for \Theta ?

Homework Equations



Ymin=n*(1-FY(y))n-1fY(y)

Also, it looks like an exponential distribution.

The Attempt at a Solution



What I need to find is E[n*Ymin]

I get [(-n)/(n+1)]/(1/ \Theta )

Is this the correct answer ?
 
Physics news on Phys.org
where did you get the eqn for Ymin it makes no sense.
 
i take that back give me a sec
 
okay here it is. the PDF of Ymin is f(y)=(-n/theta)*exp(-yn/theta). To get this we need to use the following (questionable?) sequence of logic: the probability that ymin is less than y is 1 minus the probability that ymin >= x. for this inequality to hold we need Y(i)>=x for all i. i.e. Y(1)>=x AND Y(2)>=x ... Y(n)>=x. this will give us the CDF for Ymin from which we derive the PDF of Ymin given above. Now check that E(nYmin)= theta. (I actually get
-theta but there may be some minor issue with my calculation see what you get, if
E(nYmin) =theta (not negative theta) then it is unbiased.
 
rsa58 said:
okay here it is. the PDF of Ymin is f(y)=(-n/theta)*exp(-yn/theta). To get this we need to use the following (questionable?) sequence of logic: the probability that ymin is less than y is 1 minus the probability that ymin >= x. for this inequality to hold we need Y(i)>=x for all i. i.e. Y(1)>=x AND Y(2)>=x ... Y(n)>=x. this will give us the CDF for Ymin from which we derive the PDF of Ymin given above. Now check that E(nYmin)= theta. (I actually get
-theta but there may be some minor issue with my calculation see what you get, if
E(nYmin) =theta (not negative theta) then it is unbiased.

Hello,

I don't really understand what you are trying to do here.
you found the pdf for Ymin ? we are given the pdf
Also, I got the CDF for Ymin, but am not sure I did it the right way.
And , I don't really understand what you got for the CDF..

Thanks.
 
You are given the distribution for the individual values - it is the distribution from which you take the sample. The minimum is a statistic, and you need its distribution.

The derivation outlined above is correct. If Y_{\min} is the minimum of a sample, then to get its distribution (assuming the sample comes from a continuous distribution, as yours does). I'll use G, g for the CDF and PDF of Y_{\min}, and F, f for the CDF and PDF of the population.

<br /> P(Y_{\min} \ge t) = P(X_1 \ge t \text{ and } X_2 \ge t \dots \text{ and } X_n \ge t) = (1-F(t))^n <br />

because of independence, so the CDF of Y_{\min} is

<br /> G(t) \equiv P(Y_{\min} \le t) = 1 - (1 - F(t))^n<br />

and the density Y_{\min}is

<br /> g(t) = G&#039;(t) = n(1-F(t))^{n-1} f(t)<br />

In your problem the original data come from an exponential distribution. Use the CDF for that in place of F, the PDF in place of f, to get the density of Y_{min} in this particular case. The expected value of the minimum is

<br /> \int t g(t) \, dt<br />

and this will be a function of \theta. You should be able to finish the problem from there.
 
Question: A clock's minute hand has length 4 and its hour hand has length 3. What is the distance between the tips at the moment when it is increasing most rapidly?(Putnam Exam Question) Answer: Making assumption that both the hands moves at constant angular velocities, the answer is ## \sqrt{7} .## But don't you think this assumption is somewhat doubtful and wrong?

Similar threads

  • · Replies 2 ·
Replies
2
Views
573
  • · Replies 5 ·
Replies
5
Views
2K
  • · Replies 11 ·
Replies
11
Views
2K
  • · Replies 1 ·
Replies
1
Views
4K
Replies
9
Views
4K
  • · Replies 1 ·
Replies
1
Views
2K
Replies
3
Views
3K
  • · Replies 2 ·
Replies
2
Views
1K
  • · Replies 4 ·
Replies
4
Views
2K
  • · Replies 2 ·
Replies
2
Views
5K